Download as pdf or txt
Download as pdf or txt
You are on page 1of 21

CHAPTER

15 ELECTROMAGNETIC INDUCTION

 1 R (b) E rms = 402 + (40 − 10)2 = 50 V


509.   = …(1)
 2 R + 2
X c2 E peak = E 0 = 50 2 V
 3 R + 10
  = …(2) (c) ωL i rms = 40
 
2 (R + 10)2 + X c2 40 1
L= = H
Solving these two equations we get 10 × 100π 25π
R = 5 ohm. i rms 10 1
= 10, C = = F
510. Applying Kirchhoff’s law in loop abcdefa ωC 100π × 10 100π
 di 
E − L 1  − i1R2 = 0 512. Current before closing the switch
 dt 
V
Ldi1 i0 =
i1 t
R0 + R
∫0 E − i1R2
= ∫0 dt
Let current in the circuit is i at any time t.
E   t  Applying KVL in the circuit
⇒ i1 = 1 − exp  − R2  
R2   L  di
a b i1 c
V=L + Ri
dt
i
i2 di Ri V
⇒ + =
L dt L L
R1
2 On solving we get,
R
V − t
S R2
i= + Ce L
f d R
e
V
Potential drop across inductor is at t = 0 i = i0 =
R0 + R
di  t
L 1 = E exp  − R2  = 10e −5t volts C=−V
R0
dt  L so,
R(R0 + R)
E − Rt
in steady state i1 =
R2 V VR0e L
i= −
When the switch S is opened, R R(R + R0)
E  R  t 1
i =   exp −  1 + R2  513. (a) I (t) = (E1 + E 2)
 R2    2  L R

Putting the values, we get, i = 5 e −10t 25 3 (1 + 2 sin ωt)


=
(direction of current is from c to d) R
40 Heat produced in one cycle of AC
511. (a) i rms = = 10 A 2π / ω
4 = ∫0 I 2 (t) Rdt
i peak = 10 × 2 = 10 2 A

More: WWW.JEEBOOKS.IN
Solutions 377
(25 3)2
100 Ω
R
2π / ω +
= ∫0 (1 + 2 sin 2 ωt + 2 2 sin ωt) dt
10V
10 mF

25 × 25 × 3  2π 2π  3
=  ω + ω  = 2 J
50 q
E − IR − =0
Number of cycles in 14 minute is c
t 1 q dq
N = 14 × 60 × 50 ⇒ ∫0 RC
dt = ∫Q0 EC − q
Total heat produced
3 q = 50 (2 − e −1) mC, at t = 1 sec
Q = × 14 × 60 × 50 J
2 q = 50 (2 − e −1) = 1.63 × 50 = 81.5 mC
= 63000 J Voltage across capacitor at that time
(b) Q Q = ms ∆ t q 81.5 × 10−3
V= =
⇒ T f − Ti =
Q
=
63000
= 5° C C 10 × 10−3
ms 3 × 4200
= 8.15 volt
T f = 5 + Ti = 25° C 1 2 1
(b) Limax = CVmax
2
2
(c) I DC Rt = Q 2 2
Q 63000 C
⇒ I DC = = = 1.22 A ∴ imax = ⋅ Vmax
Rt 50 × 14 × 60 L
R VR 80 10
514. (a) cos θ = = = = × 8.15 = 16.3 A
Z V V 2.5
80 1 1000
0.8 = Frequency = =
V 2π LC 2 π × 25
80 103 20
V= = 100 volt = Hz = Hz
0.8 50π π
(b) Q V 2 < VR2 + VC2 516. Let v be the velocity of rod at any time t down
(where VR = iR = 80 volt) the plane.
The motional emf e = Bbv ⊥
∴ Nature of element in box will be inductive. (here v ⊥ is the component of velocity
∴ V 2 = VR2 + (VC − VL )2 perpendicular to magnetic field and length of
conductor both, which is v = v cos α
So, either VC − VL = 60 volt or equal to v cosα in this α
VL − VC = 60 volt. Therefore VL is either 40 case) v
volt or 160 volt. Since i = 1 A. Hence X L is ∴ e = Bbv cosα
α
either 40 ohm or 160 ohm. dq
∴ 2π (50) L = 40 or 160 current in the circuit i =
dt
 0.4   1.6 where q = eC = CBbv cos α
∴ L=  henry or   henry
 π   π   dv 
∴ i = (CBb cos α)   = (CBb cos α) ⋅ a
515. (a) Initial charge = CV0 = Q0  dt 

= 10 × 10−3 × 5 = 50 mc where ‘ a’ is the acceleration of the rod. This


current i is perpendicular to paper inwards as
When capacitor is connected at position 1. shown in figure.

More: WWW.JEEBOOKS.IN
378 Problems in Physics
Fm cosα = ibB cosα
So, it is a case of charging of a capacitor in
i
α C – R circuit. Hence
Fm = ibB
mg sinα q = q0 (1 – e – t / τ C ) …(2)
µ li C ln (2)
α where q0 = e t C = 0 0
2π t 0
The rod is acted upon by the force of gravity and τ C = CR
and magnetic force. Let us write the equation
of motion for the rod : substituting in (2) we get
ma = mg sin α – ibBcosα µ li C ln ( 2)
q= 0 0 (1 – e– t/CR )
= mg sin α – (CB 2b2 cos2 α) a 2π t 0
mg sin α (b) Current at any time t is
or a = = constant …(1) dq e t – t/CR
m + CB b cos α 2 2 2
i= = e
dt R
2l  1 2
(a) t = l = at  ∴ Heat generated in resistance
a  2  t t
e2
2l H = ∫ i 2R dt = ∫ Rt2 e
– 2t / CR
R dt
or t = ( m + Cb 2 B 2 cos 2 α)
mg sin α 0 0

e t2C
(b) v = 2al (v 2 = 2al ) H = (1 – e – 2t/CR ) …(3)
2
2 mgl sin α
or v = Energy stored in capacitor
m + Cb 2 B 2 cos 2 α 1 q2 1 2
U= = e t C (1 – e – t / CR )2 …(4)
517. (a) Magnetic flux passing through an element 2 C 2
shown in figure is ∴ The desired ratio is
dx H 1 − e–2 t/ CR
R =
C l U (1 – e– t/ CR ) 2
i
q
(c) V0 = = e t (1 – e – t / CR )
C
x  et 
So t = CR ln  
l l  et – V0 
µ0i µ 0 li 0 ln ( 2)
dφ = Bldx = l ⋅ dx where et =
2π x 2π t 0
2l
µ 0 il dx 518. In the figure OA = x ,
∴ Total flux φ = ∫ 2π x
l B
C
µ il
∴ φ = 0 ln (2) O F
2π θx A θ = 60°
µ 0 l  i0  i0  D
=  ln (2) ⋅ t i = t
 2π  t0  t0 
Magnitude of induced emf 2x
then OD = OC =
dφ µ 0 l i 0 ln (2) 3
et = = …(1)
dt 2π t 0 and AD = AC =
x
= constant 3

More: WWW.JEEBOOKS.IN
Solutions 379
So length of the conducting rod DC is  E  1
∴ P3 = (Li)    
2x  L  e
l = 2 AD =
3 Ei 5 × 0.316
2 = = = 0.582 watt …(3)
∴ motional emf e = Bvl = Bvx e 2718
.
3 From equations (1), (2) and (3) we can see
Resistance in the circuit R = ρ (OD + OC) that
4
or R = 2ρ (OD) = ρx P1 = P2 + P3
3
or the power delivered
∴ current in the circuit = power dissipated as heat + power stored
e Bv It is same as required by the principle of
i= = (anticlockwise)
R 2ρ conservation of energy.
∴ magnetic force on the rod 520. Current in the circuit at time t = 0 is
 Bv   2x  E 20
i= = = 4A (clockwise)
Fm = ilB =     (B)
 2 ρ  3  R 5
 B 2v  ∴ magnetic force on the rod is
or Fm =   x (leftwards) Fm = ilB = (4) (10) (0.5) = 20 N (upwards)
 3ρ
weight of the rod
∴ External force applied is
W = mg = (1) (10) = 10 N (downwards)
 B 2v 
F =  x (rightwards) Since Fm > W , the rod will move upwards
 3ρ
Let v be the velocity of rod at any time t
519. At any instant t, current in L – R circuit is given motional emf e = Bvl = (0.5) (v) (10) = 5 v
as net emf = E – e = 20 – 5 v
E
i = i 0 (1 – e – Rt/L ) = (1 – e – Rt / L ) 20 – 5 v 20 – 5 v
R current i = = = (4 – v)
R 5
(E = applied emf)
magnetic force
At t = time constant (L/R)
Fm = ilB = (4 – v) (10) (0.5) = (20 – 5 v)
E 5  1 
i = (1 – 1/e) = 1 –  = 0.316 A ∴ net force = Fm – W (upwards)
R 10  2718
. 
 dv 
∴ The rate at which energy is delivered by the or m   = (20 – 5 v) – (10)
battery is  dt 
P1 = Ei = (5) (0.316) watt = 1.58 watt ...(1)  dv 
or (1)   = (10 – 5 v) (m = 1kg)
At this time some energy will appear as joule  dt 
heat in the resistance, given by v t
dv
P2 = i 2R = (0.316)2 (10) = 0.998 watt or ∫ 10 – 5 v = ∫ dt
0 0
P2 = 0.998 watt …(2)
Part of the energy is stored in magnetic field, Solving this equation we get
linked with the inductor. It is given by the v = 2 (1 – e– 5 t ) m/s
relation
521. Let v be the velocity of the rod at any time t.
 di 
P3 = Li   Motional emf e = Bvl
 dt 
di E – Rt / L E 1  L q = eC = BvlC
= e = ⋅  at t =  e Bvl
dt L L e  R i1 = =
R R

More: WWW.JEEBOOKS.IN
380 Problems in Physics
i1 P i2 ( fmax = µmg ) or when
τ F ≥ τ f max
B – Taking the limiting case.
R i q C
+ τ F = τ f max or F . R = (µmg ) R
v
or F = µmg or B0 QRt = µmg
F Fm
It is given that ring starts rotating after 2
Q i2 second. So, putting t = 2 second we get
2 B0 RQ
i2 =
dq
= BlC ⋅
dv µ =
dt dt mg
Bvl dv (b) After 2 second
i = i1 + i 2 = + BlC ⋅
R dt τ F > τ f max
 v dv  Therefore, net torque is
Fm = ilB = B 2l 2  + C ⋅ 
R dt  τ = τ F – τ f max = B0QR 2t – µmgR
Net force Fnet = F – Fm 2 B0QR
Substituting µ = we get,
dv v dv  mg
or m⋅ = F – B 2l 2  + C⋅ 
dt R dt 
τ = B0QR 2 (t – 2)
dv v
or (m + B 2l 2C) = F – B 2l 2  dω 
 = B0QR (t – 2)
2
dt R or I 
 dt 
dv dt
or =  dω 
B 2l 2v m + B 2l 2C or mR 2   = B0QR (t – 2)
2
F–  dt 
R
ω 4
v t B 0Q
or ∫
dv
= ∫ m + B 2l 2C
dt or ∫ dω = m ∫2
(t – 2) dt
B 2l 2 0
0 F– ⋅v 0
R 2 B 0Q
or ω= …(1)
m
FR  B 2l 2
t
This gives v = –
Now magnetic field is switched off i.e., only
B 2 l 2 1 – e R ( m + B 2 l 2 C) 
 retarding torque is present due to friction. So,
522. (a) Magnitude of induced electric field due to angular retardation will be
change in magnetic flux is given by τf µmgR µg
α = max = =
→ → dφ dB I mR 2 R
∫ E . dl = dt = S ⋅ dt Therefore, applying
 dB  ω 2 = ω 02 – 2αθ
or E. l = πR 2(2B0 t)  = 2B0 t
 dt  2
 2 B 0Q   µg 
or 0=   – 2  θ
Here E = induced electric field due to change  m   R
in magnetic flux
2 B02 Q2R
or E (2πR) = 2πR 2 B0 t or θ=
µm 2g
or E = B0 Rt
2 B0RQ
Hence F = QE = B0QRt Substituting µ =
mg
This force is tangential to ring. Ring starts
rotating when torque of this force is greater B Q
we get θ= 0
than the torque due to maximum friction m

More: WWW.JEEBOOKS.IN
Solutions 381
523. (a) Voltage across the inductor or ma = F – ilB
VL = – L
di 2mλi 2
∴ F = ilB + (R + 2λx)
dt B 2l 2
= –(10–3) (20) volt (b) Work done by F per unit time is given by
= – 20 mV Fi (R + 2λx )
P1 = Fv = …(3)
(b) Voltage across the capacitor Bl

q 1
t and heat generated in circuit per unit time is
VC = =
C C ∫ i dt P2 = i 2 Rnet = i 2 (R + 2λx ) …(4)
0
t Therefore, fraction of work converted into
 1 
=  –6 
 10  ∫ (20 t) dt heat is
0 P i Bl
f = 2 =
20t 2 P1 F
= (106) volt = (107 t 2 ) volt
2 Substituting value of F we get
1 1 –1
(c) C VC2 > Li 2 or CVC2 > Li 2
 2 mλi ( R + 2λx ) 
2 2 f = 1 + 
 B 3l 3 
or (10–6) (107 t 2)2 > (10–3) (20 t)2
Note: The remaining part of work done goes as the
or t > 63.2 × 10–6 s change in kinetic energy of rod per time i.e.,
or t > 63.2 µs dK
=
d 1 2  dv 
 mv  = mv ⋅   = (mv ) (a)
dt dt  2   dt 
524. (a) Suppose at any time t, rod MN is at a
distance x from the resistance R. Velocity Substituting the values
of rod at that instant is dK 2 mλi 3
dx = (R + 2λx )2 …(5)
v= dt B 3l 3
dt
From equations (3), (4) and (5) we can see
Motional emf e = Bvl …(1) that
Total resistance Rnet of the circuit at this instant dK
is P1 = P2 +
dt
Rnet = R + 2λx 525. Let at time t angular velocity of rod be ω. Then
e Blv induced emf in the circuit is
∴ i= = (anticlockwise)
Rnet R + 2λx Bωa 2
e=
i (R + 2 λ x) 2
or v=
Bl and therefore, current in the circuit will be
Instantaneous acceleration of the rod, is e Bωa 2
dv 2λi dx 2λi i= = (radially outwards)
a= = ⋅ = ⋅v R + r 2 (R + r )
dt Bl dt Bl
Magnetic force on element dx, at a distance x
2λ i 2
or a= (R + 2λx ) …(2) from A is
B 2l 2 F = (iB ⋅ dx )
At this moment two forces are acting on the Torque on this element about A will be
rod, applied force F (towards right) and
magnetic force Fm = ilB (towards left). So dτ = F ⋅ x = (iBx ) dx
Fnet = F – Fm ∴ Total torque on the rod will be

More: WWW.JEEBOOKS.IN
382 Problems in Physics
x =a iBa 2 Current through capacitor at time t can be
τ= ∫ x = 0 dτ or τ=
2
(anticlockwise)
determined as follows :
i – ic R2
Now angular retardation is R1 C D
τ B
α= + –
I G ic q F
dω iBa 2  ma 2  i i
or –ω⋅ = I = I A = 
dθ 2 (ma 2/ 3)  3 
A H
dω 3 Bi 3 B Bωa 2
or – ω ⋅ = = E
dθ 2 m 2 m 2 (R + r )
Let the currents in different branches be i, i c
3 B 2a 2 and i – i c at any time t and q be the charge
or dω = – ⋅ dθ
4 m (R + r ) stored in the capacitor.
Applying Kirchhoff’s loop law in ABKDHA
ω 3 B 2a 2 θ
or ∫ω 0
dω = –
4 m (R + r ) ∫ 0
dθ E – iR1 – (i – i c ) R2 = 0 …(1)
in loop KDFGK
3 B 2a 2
or ω = ω0 – θ q
– (i – i c ) R2 = 0 …(2)
4 m (R + r ) C
Bωa 2 dq
But i= Also ic = …(3)
2 (R + r ) dt
Ba 2  3 B 2a 2 θ  From equation (1)
or i = ω 0 –   E + i c R2 
2 ( R + r)  4 m ( R + r)  i= 
 R1 + R2 
526. The circuit may be assumed to be equivalent
to two circuits in parallel as shown below : Substituting this in equation (2) we get
q  E + i c R2 
R2
L – – i c  R2 = 0
C  R1 + R2
R3

R1
C

L R3 q ER2  R22 
E or – = ic  – R 2
C R1 + R2 R
 1 + R 2 
+ R2
R1
 dq  R22  q ER2
or    – R 2 = –
 dt 
E
C  R1 + R2  C R1 + R2
q dq R +R t
E or ∫ 0  ER  q = R1 1 R2 2 ∫ 0 dt
2
The first is a L-R circuit. Hence current i L at  –
 R1 + R2  C
any time t will be given by
Solving this we get
i L = i 0 (1 – e – t/τ L )
  R + R2  
– 1 t
E L ECR2   CR1 R 2  
Here i0 = and τ L = q= 1– e
R3 R3 R1 + R2  
 
E  – 3 
R t
 R1 + R 2 
∴ iL = 1 – e L  dq E –  t
R3   ∴ ic = = e
CR1 R 2 
  dt R1

More: WWW.JEEBOOKS.IN
Solutions 383
Hence currents through capacitor and Substituting this value in equation (1) we get
inductor at any time t are (mgv sin θ dt) = (mv dv)
 R1 + R 2 
E –  t π 2r 4 B02a 2 v 2 cos2 θ
ic = e
CR1R 2  + ⋅ dt
R1 R
 π 2r 4 B02a 2 v cos2 θ 
E  – 3 t
R
or mg sin θ –  dt
and iL = 1 – e L   R 
R3  
 
= m ⋅ dv
527. Let v be the velocity of loop at time t and v dv t dt
v + dv in time t + dt. or ∫ =∫
0 π r B0 a cos θ
2 4 2 2 2 0 m
Distance moved by the loop in time dt, down mg sin θ – v
the plane is R
ds = v ⋅ dt mgR sin θ
∴ v= 2 4 2 2
dh = (ds) sin θ = v sin θ ⋅ dt π r B0 a cos2 θ
and dx = (ds) cos θ = v cos θ ⋅ dt  π 2 r 4 B 02 a 2 cos 2 θ t
dx  –
mR 
or = v cos θ 1 – e 
 
dt  
From conservation of mechanical energy :
g sin θ
Decreases in potential energy in time dt or v= (1 – e– Kt )
= increase in kinetic energy in time dt K
+ energy dissipated as heat in time dt π 2r 4 B02a 2 cos2 θ
Where K=
1 1  mR
i.e. (mg ) dh =  m (v + dv)2 – mv 2
 2 2  528. Power dissipated in the loop is
+ i 2R ⋅ dt e2
P= …(1)
r
 1  dv 
2
or (mgv sin θ) dt =  mv 2 1 +  where e = induced emf in the loop
 2  v
dφ dB dB
e= =S = (πa 2)
1  dt dt dt
– mv 2 + i 2Rdt
2  µ 0 i0
Here B(x ) = =B
1  dv  2π x
or (mgv sin θ) dt =  mv 2 1 + 2 
2  v  dB  µ i dx µ i v  dx 
∴   = – 0 02 ⋅ = – 0 02  = v
1   dt  2π x dt 2π x  dt 
– mv 2 + i 2Rdt (dv << v)
2  πa 2µ 0 i 0 v
∴ e= …(2)
or (mgv sin θ) dt = (mv) dv + i Rdt 2
…(1) 2π x 2
e From equations (1) and (2)
Here i=
R π 2a 4µ 20 i 02 v 2
P=
dφ dB dx 4 π 2x 4 ⋅ r
where e = = = πr 2 (B0 ⋅ a) ⋅
dt dt dt 4 Px 4r
or v=
or e = (πr B0a) (v cos θ)
2
µ 20 i 02 a 4
πr B0a v cos θ
2
2x 2
So i= or v = Pr
R µ 0 i0 a 2

More: WWW.JEEBOOKS.IN
384 Problems in Physics
529. (a) Magnetic flux linked with the loop µ 0I 0at  x + a 
φ= ln   …(1)
z
a 2π  x 
− dφ
ε=
dt
v
µ I a (x + a)
y1 = − 0 0 ln
b
2π x
µ 0I 0at  1 1  dx
y +  x + a − x  × dt
2π  
ε µ I a  x + a
I= = − 0 0 ln  
R 2π R  x 
x
µ 0I 0a 2t dx
y2 + …(2)
φ= ∫y
1
(6 − y) bdy 2πRx (x + a) dt
dx
= 6b(y 2 − y1) −
b
(y 2 − y1) (y 2 + y1) Now = 5x
2 dt
10a dx t
ab
= 6ab −[a + 2vt] ⇒ ∫x = a
x
= 5 ∫ dt
0
2 10

dφ ln 100
Induced e.m.f. = − = abv ⇒ t=
dt 5
= (0.2) (0.5) (6.5) V Putting the value of t in equation (2)
= 0.65 V I = 0.32 A
ab 531. (a) Let at any moment the speed of the
(b) Here, φ = 6ab − [a + ωt 2] dq
2 connector is v, current in the circuit is i =
where ω = acceleration dt
and charge on the capacitor is q. Then
Hence, ε (induced e.m.f.)
dv  dq
dφ m = B  l
=− = abωt dt  dt 
dt
= (0.2) (0.5) 2t = 0.2t V  Bl 
Integrating we get v =   q …(1)
m
530. Flux linked with the loop at any instant
Applying KVL in the loop we have,
B C q dq
I = i 0t + R = (E − Blv) …(2)
C dt
dq q E B 2l 2
V = 5x ⇒ + = − q
x dt RC R mR
q dq t
⇒ ∫0 E B l2 2
1 
= ∫ dt
0
− q + 
R  mR RC 
A D
dy
y E 
 − qλ 
x +a µ 0I ⇒ ln  R  = − λt,
φ= ∫x 2π y
. ady
 E 
 R 

More: WWW.JEEBOOKS.IN
Solutions 385
 B 2l 2 1  or q0 = C(Blv 0) =
CFR
…(2)
where λ= + 
 mR RC  Bl
(b) Let at some instant t, velocity of connector
E  1 − e − λt  be v then induced e.m.f. e = Blv. At this
q=  
R λ  instant, current through resistance is
E e Blv
⇒ qt → ∞ = iR = =
Rλ R R
EmC and charge on capacitor is
⇒ qt → ∞ =
(CB l + m)
2 2
q = Ce = CBlv
(b) From Eq. (1), Since v is increasing, therefore q is also
BlCE increasing or current is flowing through
vt → ∞ = dq
(m + CB 2l 2) capacitor also. This current, i c =
dt
532. (a) Due to external force F, connector starts dv
to accelerate and an e.m.f. is induced in it. or i c = CBl = CBla
dt
Due to induced e.m.f., a current flows
through it. According to Lenz’s law, force where a is acceleration of the connector.
Bil will (due to induced current) oppose Total induced current through the connector is
motion of the connector. Blv
i = iR + iC = + CBla
Steady state velocity corresponds to zero R
acceleration. It means that force Bil must Retarding force = Bil
numerically be equal to F. ∴ Net accelerating force on the connector
Let steady state velocity be v 0
= F − Bil
e = Blv 0 R
 B 2l 2v 
In case of steady state, ⇒ ma = F −  + CB 2l 2a
charge on capacitor  R 
m
remains constant, FR − B 2l 2v
therefore no current ⇒ (m + CB 2l 2)a =
B R
flows through it, hence F
dv
current flows through
C But acceleration, a =
resistance R alone. dt
∴ Induced current dv dt
∴ =
e Blv 0 FR − B l v R(m + CB 2l 2)
2 2
i= =
R R Integrating the above equation from
Retarding force (Lorentz force) (t = 0, v = 0) to (t, v)
B 2l 2v 0  
−
B 2l2  
= Bil = FR  t
 R ( m + CB l )  
2 2
R v = 2 2 1 − e 
Bl
But Bil = F  
 
B 2l 2v 0
∴ F= 533. mg − T = ma
R
FR T − Fm = ma
⇒ v0 = 2 2 …(i)
Bl mg − Fm = 2ma …(1)
In steady state charge on the capacitor is Let at any instant inductor is moving with
q0 = Ce velocity v

More: WWW.JEEBOOKS.IN
386 Problems in Physics
di 535. Let that the rod move with speed v and
Hence E−L =0
dt acceleration a along the inclined plane, while
Blvdt current I flows in it. The magnetic field brakes
= di the rod in accordance with Lenz’s law and its
L
equation of motion is
x Bl l Bl
∫0 L
dx = ∫0 di ⇒
L
x =i ma = mg sin α − BlI .
x This equation is the same in all three cases.
Now Fm = Bil = B 2l 2 The differences result from the different
L
relationships between the induced voltage
Substituting in (1) and the current flowing in the rod.
B 2l 2x dv (i) If the circuit is closed by an ohmic resistor
mg − = 2ma = 2mv
L dx R, the current I and the induced voltage
As acceleration is a function of displacement V = Blv are connected by the relationship
V Blv
x  B 2l 2x  v I= = . This shows that the braking
∫0 mg −
 L 
 dx = 2m ∫ vdv
0
R R
force increases in proportion to the speed,
with the result that the rod experiences
B 2l 2x 2 v2
⇒ mgx − = 2m decreasing acceleration and ultimately
2L 2 travels with uniform speed. This final
1/ 2
 B 2l 2x 2  maximum speed vmax can be found from
⇒ v =  gx −  the equation of motion by setting a = 0,
 2mL 
mg R sin α
vmax =
534. Let at distance x, velocity of slide wire be V. B 2l 2
Then total resistance of the closed circuit at
(ii) If the circuit is closed by a capacitor of
this instant will be
capacitance C, the relationship between
R = λ (2x + l ) the induced voltage and the current is
Motional emf e = Bvl different. The charge on the capacitor is
Bvl Bvl determined by the induced voltage and
Current i = = given by
R λ(2x + l )
Q = CV = CBlv.
Magnetic force
Note that the current flowing through the rod
B 2l 2v is equal to the time derivative of this, i. e.,
Fm = ilB =
λ(2x + l ) dQ dV dv
I= =C = CBl = CBla.
Fnet = F − Fm dt dt dt
dv B 2l 2v In other words, the current flowing in the rod is
or mv ⋅ = F0v − directly proportional to the acceleration of the
dx λ(2x + l )
rod. If the above expression for the current is
 B 2l 2  substituted into the equation of motion, the
∴ m(dv) = F0 −  dx
 λ (2x + l ) rod is found to move on the rails with uniform
acceleration
v x  B 2l 2  mg sin α
or m ∫ dv = ∫0 F0 −  dx a=
v0
 λ(2x + l ) m + B 2l 2C
F0x B 2l 2  2x + l  Induction decreases the acceleration caused
∴ v = v0 + − ln  
m 2λ m  l  by gravity by, in effect, increasing the inertial
mass of the rod. The speed of the rod and the

More: WWW.JEEBOOKS.IN
Solutions 387
charge on the capacitor are both directly Qmin
Blvmax = …(1)
proportional to the time elapsed. C
(iii) If the circuit is closed by a coil of The equation of motion of the rod is
inductance L, the relationship between dv dQ
the induced voltage and the current is m = ma = BlI = − Bl …(2)
dt dt
dI dx
L = Blv = Bl where the acceleration and the current have
dt dt
been expressed as the rates of change in
We note that, since I = 0 and x = 0 at the start velocity and charge, respectively. The
of the motion, the above formula implies that proportionality between the two rates of
the current is proportional to the x-coordinate, change holds throughout. The speed of the
LI = Blx . Substituting for the current, from this rod increases from zero to vmax , whilst the
relationship into the equation of motion, gives charge on the capacitor decreases from
B 2l 2 Q0 = CV0 to Qmin . Equation (2) can therefore
ma = mg sin α − x.
L be rewritten as
The force acting on the rod is therefore the mvmax = Bl (Q0 − Qmin )
sum of a constant term and a negative term The final velocity and the residual charge on
proportional to the displacement. This is the the capacitor can be calculated using Eqs. (1)
same as the equation of motion of a body and (2),
hung on a spring and then released. Thus, the BlCV0
rod makes harmonic oscillations about an vmax =
m + B 2l 2C
equilibrium position
mg L sin α B 2l 2C 2V0
x0 = and Qmin =
B 2l 2 m + B 2l 2C
The amplitude of the oscillation is A = x 0, and (ii) The above relations show that the
the dependence of the displacement of the rod maximum velocity of the rod is
on time is proportional to the initial voltage V0 across
x (t) = A(1 − cos ωt), the capacitor. Thus, the final kinetic
B 2l 2 energy of the rod is proportional to V02 (for
where ω2 = given values of C and m), i. e.,
mL
proportional to the initial energy of the
536. (i) At the instant when the capacitor is
V system. The coefficient of proportionality
connected, a current I = 0 starts flowing can be regarded as the efficiency η of the
R apparatus (considered as an electro-
in the rod, which experiences a force magnetic gun), and can be written in the
F = BlI and an initial acceleration
form
BlV0
a= . In accordance with Lenz’s law, 1 2
mR mvmax
2 1
the voltage induced in the moving rod η= = 2
1  m
causes the current flowing in the rod to CV0 2
Bl C 
2  + 
decrease. The charge Q on the capacitor  Bl C m 
decreases and consequently so does the
voltage across it. Meanwhile the voltage The product of the two terms in the brackets is
induced in the rod increases, until the two 1, and from the inequality between arithmetic
voltages cancel each other out. The rod and geometric means, it follows that their sum
then continues with its maximum velocity is at least 2. This means that the efficiency of
given by the electromagnetic gun cannot be more than
25 per cent.

More: WWW.JEEBOOKS.IN
388 Problems in Physics
Note: If the condition for maximum efficiency Inserting this and the value of B into equation
m = CB 2 l 2 is substituted into the expression for (1), we get
the final charge on the capacitor we find that
VC
Qmin = 0 , i. e. , only half of the initial charge on
m (2R 2 + r 2) µ 0nk q
t = qµ 0nkt,
2 r r 2 m
the capacitor is left. Thus, only one-quarter of
the initial energy of the capacitor is left; one- which is satisfied if
quarter of it is transformed into the kinetic (2R 2 + r 2)
energy of the rod, and the other half is dissipated = 1, i. e., r = 2R
in the rod as Joule heat. 2r 2
538. The total magnetic flux at the position of the
537. The current at time t is I = kt in the outer coil,
ring is made up of that due to the external
and 2I = 2kt in the inner one, where k is a magnetic field and the effects of self-
constant. Because of these currents the induction,
magnetic field in the outer coil is B = µ 0nkt,
φ = Bz πr02 + LI
whilst in the inner one it is 3B, where n is the
number of turns per unit length. The magnetic Any change in magnetic flux induces a current
flux enclosed by the particle’s trajectory of in the ring in accordance with
radius r is ∆φ
RI =
φ = π R 2 × 2B + π r 2 × B ∆t
= (2R 2 + r 2) πµ 0nkt However, this has to be zero since the ohmic
resistance of a superconducting ring is zero.
The (constant) magnitude of the induced Accordingly, the magnetic flux through the
electric field E can be calculated from the rate ring has to be constant i. e.,
of change of magnetic flux with time:
φ = B0(1 − αz ) πr02 + LI = constant

E × 2πr =
dt From the initial conditions (z = 0, I = 0), the
value of the constant is φ = B0πr02.
= (2R + r ) πµ 0nk ,
2 2
The current in the ring can be determined
(2R 2 + r 2) µ 0nk using the above equations which give
and so E=
r 2 1
I = B0απr02z
The charged particle is held in its circular orbit L
by the magnetic field, and so, from the zero The Lorentz force acting on the ring (which
net radial component of the force acting on it, can only be vertical, because of the symmetry
we obtain of the assembly) can be expressed as
mv 2 Fz = − Br I (z ) 2πr0
= qvB …(1)
r 1
= − B0βr0 B0απr02 2πr0z = − kz
The particle is accelerated along its circular L
orbit by the tangential component of the net
The Lorentz force is thus directly proportional
force according to ma t = qE, where m is the
to the vertical displacement of the ring, with
mass and q the electric charge of the particle.
the coefficient of proportionality calculable
As the magnitude of the electric field is
from the given data. (This result is only valid
constant, the speed of the particle increases
for small displacements, since the magnetic
uniformly with time,
induction is not adequately described by the
qE
v = at t = t given formulae for large ones.)
m The equation of motion of the ring is
(2R 2 + r 2) µ 0nk q ma z = Fz − mg = − kz − mg
= t
r 2 m

More: WWW.JEEBOOKS.IN
Solutions 389
This means that the ring makes harmonic 540. Inductive reactance
oscillations about the equilibrium position X L = ωL = (50) (2π) (35 × 10−3) ≈ 11 Ω
mg
z0 = − with
k Impedence Z = R 2 + X L2
z (t) − z 0 = A cos ωt,
= (11)2 + (11)2 = 11 2 Ω
k
where ω = ⋅ From the initial conditions it Given Vrms = 220 volt
m
follows that A = − z 0, and so Hence amplitude of voltage
g V0 = 2Vrms = 220 2 volt
z (t) = 2 (cos ωt − 1).
ω V0 220 2
The vertical z-coordinate is never positive, ∴ Amplitude of current i 0 = =
Z 11 2
and it follows that the Lorentz force always
points upwards, being zero at the topmost or i 0 = 20 A
point of the oscillation. The current always Phase difference
flows in the same direction around the ring. X   11 π
Substituting the numerical data gives φ = tan −1  L  = tan −1   =
 R  11 4
ω = 31.2 s−1 and A = 1 cm. The time
In L-R circuit voltage leads the current. Hence
dependence of the current flowing in the ring
instantaneous current in the circuit is,
can be expressed in terms of z (t) as
1  π
I = B0απr02z (t) i = (20 A ) sin ωt − 
L  4
1 Corresponding i-t graph is shown in figure.
= B0απr02 A(cos ωt − 1).
L V, I
v=220 2 sin ωt
The maximum value of the current, which
flows at the bottom of the oscillation, is
i=20 sin (ωt – π/4)
Imax = 39 A. 20
O T 9T/8
539. After a long time, resistance across an inductor t
–10 2 T/8 T/4 T/2
becomes zero while resistance across capacitor 5T/8

becomes infinite. Hence net external resistance,


R
+R
3R
Rnet = 2 =
2 4 541. (i) For an elemental strip of thickness dx at a
current through the batteries, distance x from left wire, net magnetic
2E field (due to both wires)
i=
3R µ I µ I
+ r1 + r2 B= 0 + 0 (outwards)
4 2π x 2π 3a − x
Given that potential across the terminals of cell µ 0I  1 1 
A is zero. =  + 
2π  x 3a − x 
∴ E − ir1 = 0
 2E  dx
or E−  r1 = 0
x
 3R/ 4 + r1 + r2 
I a I
Solving this equation we get,
4
R = (r1 − r2) a
3
3a

More: WWW.JEEBOOKS.IN
390 Problems in Physics
Magnetic flux in this strip, q = + q0 cos ωt [from Eq. (2)]
µ I 1 1  Here q0 = Ce 0
dφ = BdS = 0  +  a dx
2π  x 3a − x  µ aCI 0ω ln (2)
= 0
2a π
∴ total flux φ = ∫a dφ
The corresponding q-t graph is shown in figures.
µ 0Ia 2a  1 1  542. (a) Applying Kirchhoff’s second law :
=
2π ∫a  +  dx
 x 3a − x  dφ
− iR − L
di
=0
dt dt
µ 0Ia
or φ= ln (2) dφ di
π or = iR + L …(1)
dt dt
µ a ln (2)
φ= 0 (I 0 sin ωt) …(1) di
π This is the desired relation between i, and
dt
Magnitude of induced emf, dφ

dφ µ 0aI 0ω ln (2) dt
e= − = cos ωt
dt π (b) Equation (1) can be written as
= e 0 cos ωt dφ = iRdt + Ldi
µ aI ω ln (2) Integrating we get,
where e0 = 0 0
π ∆φ = R ∆q + Li1
Charge stored in the capacitor, ∆φ Li1
or ∆q = − …(2)
q = Ce = Ce 0 cos ωt …(2) R R
and current in the loop Here ∆φ1 = φ f − φ i
x = x0 µ 0 I0
∫ x = 2x
dq =
i= = Cωe 0 sin ωt …(3) l dx
dt 0 2π x
imax = Cωe 0 µ 0I 0l
= ln (2)

µ 0aI 0ω C ln (2)
2
= So, from Eq. (2) charge flown through the
π
resistance upto time t = T , when current is i1, is
(i) Magnetic flux passing through the square
1 µ 0 I 0 l 
loop ∆q = ln ( 2) − Li1 
R  2π 
φ ∝ sin ωt [From Eq. (1)]
i. e., magnetic field passing through the loop (c) This is the case of current decay in an L-R
is increasing at t = 0. Hence, the induced circuit. Thus
current will produce ⊗ magnetic field (from i = i 0e − t / τ L …(3)
Lenz’s law). Or the current in the circuit at t = 0 i1
will be clockwise (or negative as per the given Here i= , i 0 = i1,
4
convention). Therefore, charge on upper plate
could be written as, t = (2T − T ) = T
q L
and τL =
q0 R
+ T
i 2 Substituting these values in Eq. (3) we get :
– t
T 3T T L T
4 4 τL = =
–q0
R ln 4

More: WWW.JEEBOOKS.IN
Solutions 391
543. (i) Given R1 = R2 = 2Ω, i0 = 6A

E = 12V
L
and L = 400 mH = 0.4 H R1 i0
E
i = E = 6A R2 i=0
E L R1
R1
S steady states condition t=0
R2 S is open
(c) (d)
Two parts of the circuit are in parallel with the i
applied battery. So, the upper circuit can be
broken as : L
R1

E E L R2
R1 +
S S
R2 t=t
(e)

(a) (b)
Refer Figure (e) : Time constant of this circuit
Figure (b) is a simple L-R circuit, whose time would be
constant is L 0.4
τL ′ = = = 01
. s
τL =
L
=
0.4
= 0.2 s R1 + R2 (2 + 2)
R2 2
∴ Current through R1 at any time t is
and steady state current i = i 0e – t / τ L ′ = 6e – t / 0.1
E 12
i0 = = = 6A or i = 6e–10t A
R2 2
Direction of current in R1 is as shown in figure
Therefore, if switch S is closed at time t = 0,
or clockwise.
then current in the circuit at any time t will be
544. Magnetic field (B) varies with time (t) as shown
given by
in figure
i(t) = i 0 (1 – e – t / τ L ) B (T)
or i(t) = 6 (1 – e – t / 0. 2
) = 6 (1 – e –5 t
) =i (say)
Therefore, potential drop across L at any time
0.8
t is
di
V= L
dt t (s)
0 0.2 0.4 0.6 0.8
= L (30 e –5t ) = (0.4) (30)e –5t
dB 0.8 T
or V = 12 e– 5 t volt = =4
dt 0.2 s
(ii) The steady state current in L or R2 is Induced emf in the coil due to change in
i = 6A magnetic flux passing through it,
Now as soon as the switch is opened, current dφ dB
in R1 is reduced to zero immediately. But in L e= = NA
dt dt
and R2 it decreases exponentially. The
situation is as follows : Here A = Area of coil = 5 × 10–3 m 2;

More: WWW.JEEBOOKS.IN
392 Problems in Physics
N = Number of turns = 100 This heat is used in raising the temperature of
Substituting the values we get the coil and the water. Let θ be the final
temperature. Then
e = (100) (5 × 10–3) (4) V = 2 V
H = mw Sw (θ – 30° ) + mc Sc (θ – 30° )
Therefore, current passing through the coil Here mw = mass of water = 0.5 kg
e Sw = Specific heat of water = 4200 J/kg - K
i= (R = resistance of coil = 1.6 Ω)
R mc = mass of coil = 0.06 kg
2
or i= = 1.25 A and Sc = specific heat of coil = 500 J/kg-K
1.6
Substituting the values we get
Note that from 0 to 0.2 s and from 0.4 s to 0.6
12000 = (0.5) (4200) (θ – 30° )
s, magnetic field passing through the coil
+ (0.06) (500) (θ – 30° )
increases, while during the time 0.2 s to 0.4 s
and from 0.6 s to 0.8 s magnetic field passing or θ = 35.6 ° C
through the coil decreases. Therefore, 545. When the side EF is at a distance y from the
direction of current through the coil in these x-axis, magnetic flux passing through the loop is
two time intervals will be opposite to each X
y
other. The variation of current (i) with time (t) Y=y
E F Y
will be as shown in the figure :
i Z-direction
(A) dY
+1.25
B= ( Bay ( k
0

H G
Y=y+a
0 t (s) Y
0.2 0.4 0.6 0.8
Y=y+a B 0Y
–1.25
φ= ∫ dφ = ∫ Y = y a
(adY )

B0
φ= [(y + a)2 – y 2]
Power dissipated in the coil is 2
P = i 2R = (1.25)2 (1.6) W = 2.5 W (a) Induced emf is
– dφ B dy
Power is independent of the direction of e= = – 0 [2(y + a) – 2y]
dt 2 dt
current through the coil. Therefore, power (P)
dy
versus time (t) graph for first two cycles will be or e = B0 a
as follows dt
P e = B0av
(watt)
dy
where v = = speed of loop
dt
2.5 e B av
∴ Induced current i = = 0
R R
t (s) →
0 0.8 Direction : | B| ∝ y i.e., as the loop comes
Total heat obtained in 12,000 cycles will be down ⊗ magnetic field passing through the
H = P⋅t loop increases, therefore, the induced current
will produce magnetic field or the induced
= (2.5) (12000) (0.4)
current in the loop will be counter-
= 12000 J clockwise.

More: WWW.JEEBOOKS.IN
Solutions 393
Alternate Solution : (of part a)  B av   B (y + a)
and FGH =  0  (a)  0  (upwards)
→ →  R   a 
Motional emf in EH and FG = 0 as v || l
B02av
Motional emf in EF is = (y + a) FGH > FEF
R
 B y
e1 =  0  (a) v = B0yv (e = B/v) ∴ Net Lorentz force on the loop
 a 
B02a 2v
Similarly motional emf in E F = FGH – FEF = (upwards)
GH will be R
e1
→ B 2 a 2v $
 B (y + a) H G F =– 0
e2 =  0  (a)(v) or j
 a  e2
R
= B0 (a + y) v (c) Net force on the loop will be
Polarities of e1 and e 2 F = weight – Lorentz force (down wards)
are shown in adjoining e1 B02a 2v
e2 < e1 i
or F = mg –
figures. R
Net emf, e = e 2 – e1  dv   B 2a 2 
or m  = mg –  0  v
e = B0 a v e2  dt   R 
e B av
∴ i= = 0 dv  B 2a 2 
R R ∴ = g –  0  v = g – Kv
dt  mR 
and direction of current will be counter-
clockwise. B02a 2
where K= = constant
(b) Total Lorentz force on the loop : We mR
have seen in part (a) that induced current v t
dv dv
passing through the loop (when its speed or
g – Kv
= dt or ∫ g − Kv ∫0
= dt
is v) is 0
B av This equation gives
i= 0
R g
X v = (1 – e – Kt )
y K
B
E F B02 a 2
Here K =
mR
i
i.e., speed of the loop is increasing exponentially
with time t. Its terminal velocity will be
H G
g mgR
v
vt = = 2 2 at t → ∞
Y K B0 a
Now magnetic force on EH and FG are equal the v-t graph will be as shown.
in magnitude and in opposite directions, v
hence they cancel each other and produce no
force on the loop. vt = g/K
 B av   B y 
FEF =  0  (a)  0  (F = ilB)
 R   a 
B02avy
= (downwards) O
t
R

More: WWW.JEEBOOKS.IN
394 Problems in Physics
L = 2 mH
546. This is a problem of L-C ∴ Q = (2.0 × 10–3) (5.0 × 10–6) (22 – 12)
oscillations.
Charge stored in the Q = 3 × 10–4 C
capacitor oscillates simple or Q = 1.732 × 10 – 4 C
harmonically as
C = 5 µF 547. (i) Let v be the velocity of the rod (as well as
Q = Q0 sin (ωt ± φ) block) at any instant of time t.
Here Q0 = maximum value ofQ = 200 µC
= 2 × 10–4 C Fm

1 1
ω= =
LC (2 × 10 –3
H) (5.0 × 10–6 F)
= 104 s–1
Let at t = 0, Q = Q0 then mg
Q (t) = Q0 cos ωt …(1) Motional emf, e = BvL
dQ e BvL
i(t) = = – Q0ω sin ωt …(2) Motional current = =
dt R R
di(t)
and = – Q0ω 2 cos (ωt) …(3) and magnetic force on the rod
dt vB 2L2
Q0 Fm = iLB =
(a) Q = 100 µC or R
2
This magnetic force will be in the direction
1
At cos (ωt) = , from equation (3) : shown in figure.
2 Net force on the system at this moment will be
di  1 vB 2L2
= (2.0 × 10–4 C) (104 s–1)   Fnet = mg – Fm = mg –
dt  2 R
di
= 10 4 A/s vB 2L2
or ma = mg –
dt mR
(b) Q = 200 µC or Q0 when cos (ωt) = 1 i.e., vB 2L2
or a=g– …(1)
ωt = 0, 2π .... mR
At this time i t = – Q0 ω sin ωt Velocity will acquire its terminal value i.e.,
or i (t) = 0 [sin 0° = sin 2π = 0] v = v T when Fnet or acceleration (a) of the
particle becomes zero.
(c) i(t) = – Q0 ω sin ωt
v B 2L2
∴ Maximum value of i is Q0ω Thus 0= g – T
mR
or imax = Q0ω mgR
or vT = 2 2
= (2.0 × 10–4 C) (104 s–1) B L
imax = 2.0 A vT mgR
(ii) When v = =
(d) From energy conservation, 2 2B 2L2
1 2 1 1 Q2 then from equation (1), acceleration of the block.
Limax = Li 2 +
 mgR   B L 
2 2
2 2 2 C g
a = g –  2 2   =g –
 2B L   mR  2
or Q = LC (imax
2
– i 2)
i g
i = max = 1.0 A or a =
2 2

More: WWW.JEEBOOKS.IN
Solutions 395
i
548. (a) Consider a small element of length dx of
the rod OA situated at a distance x from O.
i0
B

A
v=
dx
x t
O

(ii) At constant angular speed, net torque = 0


The steady state current will be
Bωr 2
i = i0 =
Fig. 1 2R
Speed of this element,
t=t B
v = xω
Therefore, induced emf developed across this r/2
ω i = i0
element in uniform magnetic fied B θ θ
O t=0 O Fm
r cos θ
de = (B)(xω) dx (e = Bvl ) 2
mg
Hence total induced emf across OA,
x =r r Bωr 2
e=∫ de = ∫ Bωx dx = θ = ωt
x =0 0 2
Fig. 3 Fig. 4
Bωr 2
∴ e= From right hand rule we can see that this
2 current would be inwards (from circumference
Bωr 2 to centre) and corresponding magnetic force
(b) (i) At constant emf or P. D. e = is (Fm) will be in the direction shown in figure
2
induced across O and A. and its magnitude is given by:
The equivalent circuit can be drawn as shown B 2ωr 3
Fm = (i)(r )(B) = [Fm = ilB]
in the figure : 2R
R L
Torque of this force about centre O is
r B 2ωr 4
τ Fm = Fm ⋅ = (clockwise)
i Bωr2 2 4R
e=
2 Similarly, torque of weight (mg ) about centre
S
O is
r mgr
Fig. 2 τ mg = (mg ) cos θ = cos ωt (clockwise)
2 2
Switch S is closed at time t = 0. Therefore, it is Therefore, net torque at any time t (after
a case of growth of current in an L-R circuit. steady state condition is achieved) about
Current at any time t is given by centre O will be
i = i 0 (1 – e – t/τ L )
τnet = τFm + τ mg
e Bωr 2 B 2ωr 4 mgr
Here i0 = = = + cos ωt (clockwise)
R 2R 4R 2
L
τL = Hence the external torque applied to maintain
R a constant angular speed is
 –   t
R
B 2ωr 4 mgr
Bωr 2   
∴ i= 1– e L  τ ext = + cos ωt
2R   4R 2
 
(but in anticlockwise direction)
The i − t graph will be as follows :

More: WWW.JEEBOOKS.IN
396 Problems in Physics
π 3π But since e = BvL
Note that for <θ< , torque of weight will
2 2 e (0.6)
∴ v= = m/s = 1.0 m/s
be anticlockwise the sign of which is BL (0.6)(1.00
automatically adjusted because
cos θ = negative for π/ 2 < θ < 3π/ 2. Hence terminal velocity of the bar is 1.0 m/s
549. Let the magnetic field be perpendicular to the Power in RI is 0.76 watt
plane of rails and inwards ⊗. If v be the e2
∴ . =
076
terminal velocity of the rails, then potential R1
differences across E and F would be BvL with
e2 (0.6)2
E at lower potential and F at higher potential. ∴ R1 = = Ω = 0.47 Ω
The equivalent circuit is shown in figure (2). 076
. 076
.
B R1 = 0.47 Ω
R1 R1
A C e 2 (0.6)2
Similarly R2 = = Ω = 0.3 Ω
i1 e = B vL 1.2 1.2
E F E F R2 = 0.3 Ω
F i
R2 R2 550. Induced emfs in two loops AEFD and EBCF
B D
i2 would be
dφ  dB 
e1 = 1 = S1   = (1 × 1) (1)V = 1V
Figure-1 Figure-2 dt  dt 
e e
i1 = …(1) i2 = …(2) dφ2  dB 
R1 R2 Similarly e 2 = = S2  
dt  dt 
Power dissipated in R1 is 0.76 watt = (0.5 × 1) (1)V = 0.5V
Therefore, ei1 = 076 . watt …(3) Now since the magnetic field is increasing, the
Similarly ei 2 = 1.2 watt …(4) induced current will produce the magnetic
Now the total current in bar EF is field in ( • ) direction. Hence e1 and e 2 will be
applied as shown in the figure :
i = i1 + i 2 (from E to F)…5)
A 1Ω E 0.5 Ω i2 B
Under equilibrium condition,
i1 e2 = 0.5 V
Fm 1Ω 1Ω 1Ω
E F
i e1 = 1 V
Fg
i
1Ω 0.5 Ω
magnetic force (Fm ) on bar EF = weight (Fg ) of D i1 F i2 C
bar EF
i.e., Fm = Fg Kirchhoff's first law at junction F gives
or iLB = mg …(6) i1 = i + i 2 …(1)
From equation (6) Kirchhoff's second law in loop FEADF gives
mg (0.2)(9.8) 3i1 + i = 1 …(2)
i= = A or i = 3.27 A
LB (1.0)(0.6) Kirchhoff's second law in loop FEBCF gives
Multiplying equation (5) by e, we get 2i 2 – i = 0.5 …(3)
ei = ei1 + ei 2 = (076
. + 1.2) watt Solving (1), (2) and (3), we get
(From equations 3 and 4) i1 = (7 / 22) A
= 1.96 watt and i 2 = (6/ 22) A and i = (1/ 22) A
1.96
e= volt Therefore, current in segment AE is
i (7/22)A from E to A, current in segment
1.96 BE is 6/22 A from B to E and current in
e= V or e = 0.6 V
3.27 segment EF is (1/22) A from F to E.

More: WWW.JEEBOOKS.IN

You might also like